From 1312e13900010f02eadeaa4017ee8ac6a812d820 Mon Sep 17 00:00:00 2001 From: "W. Trevor King" Date: Fri, 27 May 2011 15:22:26 -0400 Subject: [PATCH] Add last few weeks of Phys 102 solutions. --- .../Serway_and_Jewett_8/problem24.11.tex | 19 ++ .../Serway_and_Jewett_8/problem24.13.tex | 10 + .../Serway_and_Jewett_8/problem24.21.tex | 32 +++ .../Serway_and_Jewett_8/problem24.31.tex | 60 +++++ .../Serway_and_Jewett_8/problem24.33.tex | 25 ++ .../Serway_and_Jewett_8/problem24.41.tex | 20 ++ .../Serway_and_Jewett_8/problem24.51.tex | 55 +++++ .../Serway_and_Jewett_8/problem25.07.tex | 5 + .../Serway_and_Jewett_8/problem25.12.tex | 5 + .../Serway_and_Jewett_8/problem25.16.tex | 5 + .../Serway_and_Jewett_8/problem25.21.tex | 23 ++ .../Serway_and_Jewett_8/problem25.27.tex | 56 +++++ .../Serway_and_Jewett_8/problem25.36.tex | 46 ++++ .../Serway_and_Jewett_8/problem26.04.tex | 10 + .../Serway_and_Jewett_8/problem26.16.tex | 9 + .../Serway_and_Jewett_8/problem26.18.tex | 14 ++ .../Serway_and_Jewett_8/problem26.31.tex | 8 + .../Serway_and_Jewett_8/problem26.32.tex | 17 ++ .../Serway_and_Jewett_8/problem26.57.tex | 11 + .../Serway_and_Jewett_8/problem26.67.tex | 27 +++ .../Serway_and_Jewett_8/problem27.01.tex | 18 ++ .../Serway_and_Jewett_8/problem27.17.tex | 26 ++ .../Serway_and_Jewett_8/problem27.29.tex | 21 ++ .../Serway_and_Jewett_8/problem27.31.tex | 11 + .../Serway_and_Jewett_8/problem27.32.tex | 12 + .../Serway_and_Jewett_8/problem27.49.tex | 28 +++ .../Serway_and_Jewett_8/problem28.02.tex | 12 + .../Serway_and_Jewett_8/problem28.11.tex | 22 ++ .../Serway_and_Jewett_8/problem28.21.tex | 20 ++ .../Serway_and_Jewett_8/problem28.26.tex | 14 ++ .../Serway_and_Jewett_8/problem28.59.tex | 17 ++ .../Serway_and_Jewett_8/problem28.71.tex | 223 ++++++++++++++++++ .../Serway_and_Jewett_8/problem29.02.tex | 136 +++++++++++ .../Serway_and_Jewett_8/problem29.03.tex | 57 +++++ .../Serway_and_Jewett_8/problem29.22.tex | 88 +++++++ .../Serway_and_Jewett_8/problem29.31.tex | 11 + .../Serway_and_Jewett_8/problem29.37.tex | 12 + .../Serway_and_Jewett_8/problem29.40.tex | 17 ++ .../Serway_and_Jewett_8/problem29.57.tex | 6 + .../Serway_and_Jewett_8/problem29.61.tex | 9 + .../Serway_and_Jewett_8/problem30.18.tex | 66 ++++++ .../Serway_and_Jewett_8/problem30.30.tex | 21 ++ .../Serway_and_Jewett_8/problem30.32.tex | 51 ++++ .../Serway_and_Jewett_8/problem30.61.tex | 93 ++++++++ .../Serway_and_Jewett_8/problem30.64.tex | 90 +++++++ 45 files changed, 1538 insertions(+) create mode 100644 latex/problems/Serway_and_Jewett_8/problem24.11.tex create mode 100644 latex/problems/Serway_and_Jewett_8/problem24.13.tex create mode 100644 latex/problems/Serway_and_Jewett_8/problem24.21.tex create mode 100644 latex/problems/Serway_and_Jewett_8/problem24.31.tex create mode 100644 latex/problems/Serway_and_Jewett_8/problem24.33.tex create mode 100644 latex/problems/Serway_and_Jewett_8/problem24.41.tex create mode 100644 latex/problems/Serway_and_Jewett_8/problem24.51.tex create mode 100644 latex/problems/Serway_and_Jewett_8/problem25.07.tex create mode 100644 latex/problems/Serway_and_Jewett_8/problem25.12.tex create mode 100644 latex/problems/Serway_and_Jewett_8/problem25.16.tex create mode 100644 latex/problems/Serway_and_Jewett_8/problem25.21.tex create mode 100644 latex/problems/Serway_and_Jewett_8/problem25.27.tex create mode 100644 latex/problems/Serway_and_Jewett_8/problem25.36.tex create mode 100644 latex/problems/Serway_and_Jewett_8/problem26.04.tex create mode 100644 latex/problems/Serway_and_Jewett_8/problem26.16.tex create mode 100644 latex/problems/Serway_and_Jewett_8/problem26.18.tex create mode 100644 latex/problems/Serway_and_Jewett_8/problem26.31.tex create mode 100644 latex/problems/Serway_and_Jewett_8/problem26.32.tex create mode 100644 latex/problems/Serway_and_Jewett_8/problem26.57.tex create mode 100644 latex/problems/Serway_and_Jewett_8/problem26.67.tex create mode 100644 latex/problems/Serway_and_Jewett_8/problem27.01.tex create mode 100644 latex/problems/Serway_and_Jewett_8/problem27.17.tex create mode 100644 latex/problems/Serway_and_Jewett_8/problem27.29.tex create mode 100644 latex/problems/Serway_and_Jewett_8/problem27.31.tex create mode 100644 latex/problems/Serway_and_Jewett_8/problem27.32.tex create mode 100644 latex/problems/Serway_and_Jewett_8/problem27.49.tex create mode 100644 latex/problems/Serway_and_Jewett_8/problem28.02.tex create mode 100644 latex/problems/Serway_and_Jewett_8/problem28.11.tex create mode 100644 latex/problems/Serway_and_Jewett_8/problem28.21.tex create mode 100644 latex/problems/Serway_and_Jewett_8/problem28.26.tex create mode 100644 latex/problems/Serway_and_Jewett_8/problem28.59.tex create mode 100644 latex/problems/Serway_and_Jewett_8/problem28.71.tex create mode 100644 latex/problems/Serway_and_Jewett_8/problem29.02.tex create mode 100644 latex/problems/Serway_and_Jewett_8/problem29.03.tex create mode 100644 latex/problems/Serway_and_Jewett_8/problem29.22.tex create mode 100644 latex/problems/Serway_and_Jewett_8/problem29.31.tex create mode 100644 latex/problems/Serway_and_Jewett_8/problem29.37.tex create mode 100644 latex/problems/Serway_and_Jewett_8/problem29.40.tex create mode 100644 latex/problems/Serway_and_Jewett_8/problem29.57.tex create mode 100644 latex/problems/Serway_and_Jewett_8/problem29.61.tex create mode 100644 latex/problems/Serway_and_Jewett_8/problem30.18.tex create mode 100644 latex/problems/Serway_and_Jewett_8/problem30.30.tex create mode 100644 latex/problems/Serway_and_Jewett_8/problem30.32.tex create mode 100644 latex/problems/Serway_and_Jewett_8/problem30.61.tex create mode 100644 latex/problems/Serway_and_Jewett_8/problem30.64.tex diff --git a/latex/problems/Serway_and_Jewett_8/problem24.11.tex b/latex/problems/Serway_and_Jewett_8/problem24.11.tex new file mode 100644 index 0000000..e2b4390 --- /dev/null +++ b/latex/problems/Serway_and_Jewett_8/problem24.11.tex @@ -0,0 +1,19 @@ +\begin{problem*}{24.11} +Four closed surfaces, $S_1$ through $S_4$, together with the charges +$-2Q$, $Q$, and $-Q$ are sketched in Figure P24.11. (The colored +lines are the intersections of the surfaces with the page.) Find the +electric flux through each surface. +% -2Q +% +% Q +% +% -Q + +% S_1 contains -2Q, Q +% S_2 contains Q, -Q +% S_3 contains -2Q, Q, -Q +% S_4 to the left, contains no charges +\end{problem*} + +\begin{solution} +\end{solution} diff --git a/latex/problems/Serway_and_Jewett_8/problem24.13.tex b/latex/problems/Serway_and_Jewett_8/problem24.13.tex new file mode 100644 index 0000000..2341930 --- /dev/null +++ b/latex/problems/Serway_and_Jewett_8/problem24.13.tex @@ -0,0 +1,10 @@ +\begin{problem*}{24.13} +In the air over a particular region at an altitude of $500\U{m}$ above +the ground, the electric field is $120\U{N/C}$ directed downward. At +$600\U{m}$ above the ground, the electric field is $100\U{N/C}$ +downward. What is the average volume charge density in the layer of +air between these two elevations? Is it positive or negative? +\end{problem*} + +\begin{solution} +\end{solution} diff --git a/latex/problems/Serway_and_Jewett_8/problem24.21.tex b/latex/problems/Serway_and_Jewett_8/problem24.21.tex new file mode 100644 index 0000000..f7f7656 --- /dev/null +++ b/latex/problems/Serway_and_Jewett_8/problem24.21.tex @@ -0,0 +1,32 @@ +\begin{problem*}{24.21} +Figure P24.21 represents the top view of a cubic gaussian surface in a +uniform electric field \vect{E} oriented parallel to the top and +bottom faces of the cube. The field makes an angle $\theta$ with side +$a$, and the area of each face is $A$. In sumbolic form, find the +electric flux through \Part{a} face $a$, \Part{b} face $b$, \Part{c} +face $c$, \Part{d} face $d$, and \Part{e} the top and bottom faces of +the cube. \Part{f} What is the net electric flux through the +cube? \Part{g} How much charge is enclosed within the gaussian +surface? +% ^ +% a E\th| +% d b \ | +% c \| + +\end{problem*} + +\begin{solution} +\Part{a} + +\Part{b} + +\Part{c} + +\Part{d} + +\Part{e} + +\Part{f} + +\Part{g} +\end{solution} diff --git a/latex/problems/Serway_and_Jewett_8/problem24.31.tex b/latex/problems/Serway_and_Jewett_8/problem24.31.tex new file mode 100644 index 0000000..7f75ff2 --- /dev/null +++ b/latex/problems/Serway_and_Jewett_8/problem24.31.tex @@ -0,0 +1,60 @@ +\begin{problem*}{24.31} +A solid sphere of radius $40.0\U{cm}$ has a total positive charge of +$26.0\U{$\mu$C}$ uniformly distributed throughout its volume. +Calculate the magnitude of the electric field \Part{a} +$0\U{cm}$, \Part{b} $10.0\U{cm}$, \Part{c} $40.0\U{cm}$, and \Part{d} +$60.0\U{cm}$ from the center of the sphere. +\end{problem*} + +\begin{solution} +\Part{a} +The electric field at $0\U{cm}$ must be zero by symmetry. Because the +situation is symmetric, there is no preferred direction for the +electric field. + +\Part{b} +The charge density inside the sphere is +\begin{equation} + \rho = \frac{Q}{V(R)} = \frac{Q}{\frac{4}{3}\pi R^3} \;. +\end{equation} + +Inside the sphere, the enclosed charge is +\begin{equation} + q_\text{in} = \rho V(r) = \frac{Qr^3}{R^3} \l. +\end{equation} + +By Gauss's law, the electric flux is +\begin{equation} + \Phi_E = \frac{q_\text{in}}{\epsilon_0} = \frac{Qr^3}{\epsilon_0 R^3} \;, +\end{equation} +which is uniformly distributed over the surface of the gaussian +sphere, so +\begin{equation} + E = \frac{\Phi_E}{A} + = \frac{\frac{Qr^3}{\epsilon_0 R^3}}{4\pi r^2} + = \frac{Qr}{4\pi\epsilon_0 R^3} \;. + = \frac{kQr}{R^3} \;. +\end{equation} +The electric field at $r=10.0\U{cm}$ is therefore +\begin{equation} + E(r=10.0\U{cm}) = \ans{365\U{kN/C}} +\end{equation} + +\Part{c} +On the surface of the sphere, the entire charge is enclosed, so +\begin{equation} + E = \frac{kQ}{r^2} \;. +\end{equation} +For this particular $r$, +\begin{equation} + E(r=40.0\U{cm}) = \ans{1.46\U{MN/C}} \;. +\end{equation} + +\Part{d} +Further outside the surface, we still enclose the same total charge, so +\begin{equation} + E(r=60.0\U{cm}) = \ans{649\U{kN/C}} \;. +\end{equation} + +All of these electric fields are directed radially outward. +\end{solution} diff --git a/latex/problems/Serway_and_Jewett_8/problem24.33.tex b/latex/problems/Serway_and_Jewett_8/problem24.33.tex new file mode 100644 index 0000000..817e8d4 --- /dev/null +++ b/latex/problems/Serway_and_Jewett_8/problem24.33.tex @@ -0,0 +1,25 @@ +\begin{problem*}{24.33} +Consider a long, cylindrical charge distribution of radius $R$ with a +uniform charge density $\rho$. Find the electric field at distance +$r$ from the axis, where $r0$) and down +through the center wire ($I_2<0$) as the capacitor discharges. +Repeating our Kirchhoff loop from \Part{b}, +\begin{align} + 0 &= - I_2 R_2 - \frac{q}{C} + I_3 R_3 \\ + \frac{q}{C} &= (R_2+R_3)I_3 = R'I_3 \;, +\end{align} +where $R'\equiv R_2+R_3$ is shorthand to allow cleaner formulas for +the rest of the problem, and $q$ is the charge on the capacitor +($q(t=0)=Q$). + +As the capacitor discharges, $\deriv{t}{q}<0$, so $I_3=-\deriv{t}{q}$. +We can then solve for $q(t)$ by integrating with respect to time. +\begin{align} + \frac{q}{C} &= -R'\deriv{t}{q} \\ + \frac{-\dd t}{R'C} &= \frac{\dd q}{q} \\ + \int_{t=0}^t \frac{-\dd t}{R'C} &= \int_{t=0}^t \frac{\dd q}{q} \\ + \frac{-t}{R'C} &= \ln(q) - \ln(Q) = \ln\p({\frac{q}{Q}}) \;, +\end{align} +where $-\ln(Q)$ is a constant of integration which is determined by +the conditions at $t=0$. Raising $e$ to either side this equation, we +have +\begin{align} + e^{\frac{-t}{R'C}} &= e^{\ln\p({\frac{q}{Q}})} = \frac{q}{Q} \\ + q &= Q e^{\frac{-t}{R'C}} \\ + I_3 &= -\deriv{t}{q} + = -\p({\frac{-1}{R'C}}) Q e^{\frac{-t}{R'C}} + = \frac{V_0}{R'} e^{\frac{-t}{R'C}} \;, +\end{align} +where $V_0=Q/C$ is the initial voltage across the capacitor. You can +see that both the charge on the capacitor and the current through the +loop will drop off exponentially with a time constant $R'C$ as the +system discharges. + +Now we can put together our knowledge of the switch-closed and +switch-open cases to write an equation for $I_2$. + +\begin{equation} + I_2 = \begin{cases} + \frac{-V}{R_1+R_2} = \ans{-333\U{$\mu$A}} + & t<0 \\ + -\frac{Q}{R'C} e^{\frac{-t}{R'C}} + = \frac{C I_2(t<0) R_2}{R'C} e^{\frac{-t}{R'C}} + = \frac{-V R_2}{(R_1+R_2)\cdot(R_2+R_3)} e^{\frac{-t}{R'C}} + = \ans{-(278\U{$\mu$A})\cdot e^{\frac{-t}{180\U{ms}}}} + & t>0 + \end{cases} +\end{equation} + +\Part{d} +Plugging into our formula for $q(t)$ +\begin{align} + \frac{Q}{5} &= q(t) = Q e^{\frac{-t}{R'C}} \\ + \frac{1}{5} &= e^{\frac{-t}{R'C}} \\ + \ln\p({\frac{1}{5}}) &= \frac{-t}{R'C} \\ + t &= -R'C\ln\p({\frac{1}{5}}) = R'C\ln(5) = \ans{290\U{ms}} +\end{align} +\end{solution} diff --git a/latex/problems/Serway_and_Jewett_8/problem29.02.tex b/latex/problems/Serway_and_Jewett_8/problem29.02.tex new file mode 100644 index 0000000..7b00983 --- /dev/null +++ b/latex/problems/Serway_and_Jewett_8/problem29.02.tex @@ -0,0 +1,136 @@ +\begin{problem*}{29.3} +Determine the initial direction of the deflection of charged particles +as they enter the magnetic fields shown in Figure~P29.2. +\begin{center} +\hspace{\stretch{1}} +\begin{asy} +import Mechanics; +import ElectroMag; + +real u = 1cm; +real height = 2u; +real width = 2u; +real dx = 0.49u; +real dy = dx; + +Vector Bs[]; +int n = (int)(width / dx); +int m = (int)(height / dy); +real xstart = -width/2 + (dx+fmod(width,dx))/2.0; +real ystart = -height/2 + (dy+fmod(height,dy))/2.0; +for (int i=0; i0$ in \cref{eq:30.61.a} means the magnetic field points +\ans{out of the page}. + +\Part{c} +We take the same approach for point $Q$, $30.0\U{cm}$ above the +intersection, but using the right hand rule to determine the direction +of the magnetic field contributions, we can see that we'll need to use +vector addition to determine the resulting magnetic field. +\begin{center} +\begin{asy} +import Mechanics; +import ElectroMag; + +real d = 0.5cm; + +Vector Bx = BField(mag=5d, dir=-90, "$\vect{B}_{xP}$"); +Vector By = BField(mag=3d, dir=0, "$\vect{B}_{yP}$"); +pair b = Bx.mag*dir(Bx.dir) + By.mag*dir(By.dir); +Vector B = BField(mag=length(b), dir=degrees(b), "$\vect{B}_P$"); +Angle t = Angle(dir(0), (0,0), b, "$\theta_P$"); +t.draw(); +Bx.draw(); +By.draw(); +B.draw(); +dot("$P$", (0,0), W); + +draw_ijhat((-3d, -2.5d)); +\end{asy} +\end{center} +The contribution from the $x$-aligned wire will be along $-\jhat$, +while the contribution from the $y$-aligned wire will be along +$\ihat$. The total magnetic field is +\begin{align} + \vect{B}_P + &= \frac{\mu_0 I_x}{2\pi r} \cdot (-\jhat) + + \frac{\mu_0 I_y}{2\pi r}\ihat + = \frac{\mu_0}{2\pi r}\cdot(I_y\ihat-I_x\jhat) + = (2.00\ihat - 3.33\jhat)\U{$\mu$T} \\ + |\vect{B}_P| &= \sqrt{2.00^2 + (-3.33)^2}\U{$\mu$T} + = \ans{3.89\U{$\mu$T}} \\ + \theta_P &= \arctan\p({\frac{-3.33}{2.00}}) + = \ans{-59.0\dg} \;. +\end{align} +\end{solution} diff --git a/latex/problems/Serway_and_Jewett_8/problem30.64.tex b/latex/problems/Serway_and_Jewett_8/problem30.64.tex new file mode 100644 index 0000000..7cee9e8 --- /dev/null +++ b/latex/problems/Serway_and_Jewett_8/problem30.64.tex @@ -0,0 +1,90 @@ +\begin{problem*}{30.64} +Two coplanar and concentric circular loops of wire carry currents of +$I_1=5.00\U{A}$ and $I_2=3.00\U{A}$ in opposite directions as in +Figure P30.64. If $r_1=12.0\U{cm}$ and $r_2=9.00\U{cm}$, what +are \Part{a} the magnitude and \Part{b} the direction of the magnetic +field at the center of the two loops? \Part{c} Let $r_1$ remain fixed +at $12.0\U{cm}$ and let $r_2$ be variable. Determine the value of +$r_2$ such that the net field at the center of the loops is zero. +\begin{center} +\begin{asy} +import Mechanics; +import ElectroMag; +import Circ; + +real u = 0.2cm; +real r1 = 12u; +real r2 = 9u; +real dr = 6pt; + +draw(scale(r1)*unitcircle, line); +draw(scale(r2)*unitcircle, line); +Distance d1 = Distance((0,0), r1*dir(-45), "$r_1$"); +d1.draw(); +Distance d2 = Distance((0,0), r2*dir(45), "$r_2$"); +d2.draw(); + +draw(arc((0,0), r1+dr, angle1=10, angle2=-10), CurrentPen, ArcArrow); +label("$I_1$", (r1+dr)*dir(0), E); +draw(arc((0,0), r2-dr, angle1=170, angle2=190), CurrentPen, ArcArrow); +label("$I_2$", (r2-dr)*dir(180), E); +\end{asy} +\end{center} +\end{problem*} + +\begin{solution} +\Part{a} +We're interested in the magnetic field generated at the center of the +ring. There is no path for an Amperian loop that takes advantage of +the problem's symmetry, so we'll use the Biot--Savart law directly. +Integrating around a single ring of radius $r$, the magnetic field at +the center is given by +\begin{align} + \vect{B} &= \oint_S \dd\vect{B} + = \oint_S \frac{\mu_0}{4\pi}\cdot\frac{I \dd\vect{s} \times \rhat}{r^2} + = \frac{\mu_0 I}{4\pi r^2} \oint_S |\dd\vect{s}|\cdot|\rhat|\cdot\sin(\theta) + = \frac{\mu_0 I}{4\pi r^2} \oint_S |\dd\vect{s}| + = \frac{\mu_0 I}{4\pi r^2} \cdot 2\pi r + = \frac{\mu_0 I}{2 r} \;, +\end{align} +where we took advantage of the following facts: +\begin{itemize} + \item The current $I$ and distance $r$ from the wire to the point we + care about are the same for every segment $\dd\vect{s}$ in the + loop, so we can pull these constants outside the integral. + \item The cross product of two vectors can be written + $\vect{A}\times\vect{B}=|\vect{A}|\cdot|\vect{B}|\cdot\sin(\theta)$. + \item \rhat is a unit vector ($|\rhat|=1$). + \item \rhat always points centerward and $\dd\vect{s}$ is always + tangent to the ring, so $\theta=90\dg$ and $\sin(\theta)=1$. + \item $\oint_S |\dd\vect{s}|$ is just the total length of the path + $S$, which is the circumfernce of the ring ($2\pi r$). +\end{itemize} +Using the right hand rule, it is clear that a counterclockwise current +will generate a magnetic field at the center of the ring which points +out of the page. + +Now that we have a formula for the magnetic field at the center of the +ring, we can calculate the magnetic field at the center of the two +rings using superposition. Letting \emph{out of the page} be the +positive direction, +\begin{equation} + B = \frac{\mu_0 (-I_1)}{2 r_1} + \frac{\mu_0 I_2}{2 r_2} + = -5.24\U{$\mu$T} \;, \label{eq:30.64.a} +\end{equation} +so the magnetic field has a magnitude of $\ans{5.24\U{$\mu$T}}$. + +\Part{b} +Because we picked \emph{out of the page} as the positive direction, +the $-$ sign in \cref{eq:30.64.a} means that the magnetic field points +\ans{into the page}. + +\Part{c} +If we allow $r_2$ to vary, we have no magnetic field when +\begin{align} + 0 &= B = \frac{\mu_0 (-I_1)}{2 r_1} + \frac{\mu_0 I_2}{2 r_2} \\ + \frac{\mu_0 I_1}{2 r_1} &= \frac{\mu_0 I_2}{2 r_2} \\ + \frac{I_1}{r_1} &= \frac{I_2}{r_2} \\ + r_2 &= \frac{I_2}{I_1} r_1 = \ans{7.20\U{cm}} \;. +\end{align} +\end{solution} -- 2.26.2